- Joined
- Apr 2025
- Subscription
- Free
I was torn between C and E.
Got the wrong answer C.
Teaching is different from setting curricula!
If it was said "Academics are not capable of settings curricula suitable for relevant management training," it would have been a correct answer choice.
Again, CAREFUL READING, and DETAIL MATTERS.
It would have been very helpful if I knew what opinions and transcripts meant in the legal context. I find Law topic more difficult than other topics.
The "without" used here isn't a logical indicator.
It is more like saying one can X without Y (X and /Y)
So the trick here is the use of the word "can" which is not a logical indicator.
When there is a "probability indicator (can, could, may, etc.)" and a familiar logical indicator, be careful not to confuse it as a logical indicator.
The premise CGOB & /C
----------------------------------------------------
The conclusion I & /C
Necessary Assumption is then CGOB → I
Didn't understand what "the outcome of the study" meant by Engle.
I mistakenly interpreted "the outcome of the study" to be "failure or success of the study = can the study be performed or not?" instead of "drug effective or not"
Because to me, confusing the drug's effectiveness with the side effect is so unreasonable, and couldn't even imagine someone would actually do that.
Now I know the people in LSAT questions could be super unreasonable and anything is possible.
Few can be translated into: some & Most Not
Few cowards fail to be fools.
Some cowards fail to be fools. = Some cowards are not fools.
&
Most cowards don't fail to be fools. = Most cowards are fools.
Completely fooled by the grammar of AC (A).
I thought it as:
People ‑m→ Need Love&Support
/D or Happy → Love & Support
-------------------------------------------------------------------------
More Love & Support ---------------- More Happy
Looking at it like this, I realize the most relationship is just a throwaway bit. Dang.
I'm glad that I encountered this type of flaw in the course because this type of flaw was not registered to my thought process.
To get Veg corr 50HD, we need more than Veg ‑m→ 50HD
What we need to know is the other side of the data Veg ‑m→ 50HD
Saying that most people of X group shows Y attribute does not mean there is a correlation between X and Y variable.
The above statement turned into data is like:
Vegetarian / Heart Disease
Yes / No
Yes / No
Yes / No
Yes / Yes
Yes / No
You see, the column containing +50 Vegetarian is all YES, and from this, all we can know about is % of vegetarians with heart disease.
While what we need instead is something like this:
Vegetarian / Heart Disease
Yes / No
No / Yes
No / No
No / Yes
Yes / No
Yes / No
Yes / No
Yes / No
No / No
With the above data, we can start to say if there is any meaningful correlation between variable 1 (vegetarian or not) and variable 2 (heart disease or not).
Recap: To evaluate a correlation, we need two independent variables, each of them having at least two data points.
This question could have been much easier to solve if I focused more on actual logic, less on the relationship.
I initially looked at the cause and effect relationship: cause - CO2 increase; result - PS increase
Then Burning Fossil Fuel and Industrial Activity (BFFIA) contributes to CO2 increase
PS increase is beneficial for agriculture.
So the overall relationship I came up with was the following:
BFFIA contributes to CO2 increase results PS increase (is beneficial for Agriculture - this part is assumed by the author, which is not unreasonable)
Therefore BFFIA is beneficial for Agriculture.
With this relationship, I tried to find X contributes to Y → Z (is good for A); therefore, X is good for A.
But the flaw was in the word PURELY, which committed a grave mistake that only focuses on one outcome while ignoring any other possibilities.
How sneaky the LSAT writers hide some statements!
There are wild boars that are a kind of pig.
There are edible pigs that are cute.
Therefore, some wild boars are cute and edible.
Something Could happen /= something Must happen.
Sufficiency and Necessacity confusion often means someone made a mistake taking a necessary condition as a sufficient condition.
When a necessary condition is confused as a sufficient condition, we are in the territory of the arguer neglecting a possibility that it Can be "N" And Not "S"
Penguin → Bird
A creature (i.e. crow) can be a bird And Not be a penguin.
Me before: oh, sufficiency necessity confusion! (blindly looking for the AC exactly saying this and confused when I don't see one. panic. can't process words in my head. get's the answer wrong.).
Me now: oh, sufficiency necessity confusion! you idiot is neglecting the case that it Can be Necessary And Not be Sufficient! (but still gets the answer wrong for not reading carefully)
NEVER CHANGE means Can't add new things...
X: PA tells everything to everybody.
Y: Reporters know exactly the same amount of information
Z: Reporter can't scoop
X → Y → Z
/X
---------------------
/Z
But this is invalid because negating sufficient condition doesn't do anything. It still leaves Y → Z relationship intact.
So the error is that Y still stands as a sufficient condition and that's why we can't say /Z.
My bias (fear against a heart attack) tricked me up on this.
I thought, "well, I don't want to die of a heart attack after being diagnosed as not having a heart attack" in a sense that a false positive is better than not being diagnosed having a heart attack.
But that's not true if I get immediately hospitalized with a false positive and later left with a huge hospital bill that was completely unnecessary. In such a case, the computer's false positive will give me a real heart attack, so not great.
Mechanically removed B because of "some".
Never do that.
Ahh, I thought K can't vote for M because K and M have zero disagreement, and my focus was on "at least 1 disagreement." My thought was that K doesn't have any disagreement with M so she can't vote for M.
But K can vote for M because the principle is that she can vote for the candidate who has the least number of disagreement, and zero disagreement is less than 1 (L & M).
Acceptable to vote for this candidate → Disagree less with this candidate than any other candidates
But this question sucks.
If we look at the entire conditional chain spatially, it is more clear to understand.
We first have this clearly sequential chain between the three trees, and most of the Sycamores overlap with Tulips, and there also must be some Sycamores that are not as old as than the youngest Tulips (see where "es" falls in the diagram below).
Tulips | Maples | Dogwoods
Sycamores
(the tail end of Sycamores is not determined)
And C is saying, "Some Sycamores are not as old as the oldest Dogwoods" but this doesn't have to be true, as we can see from the diagram above. Sure, you can have a diagram like the following:
Tulips | Maples | Dogwoods
Sycam o r e s
Here, "s" falls in the age that is "not as old as the oldest Dogwoods."
but does this have to be true? No, because again, the tail end of Sycamores is not determined.
It took me a while to figure out this diagram, and I got it wrong during the timed session.
I hope this makes sense!
An analogy makes the problem a little more to understand.
The world
I have a Good Dog (GD) and a Bad Dog (BD). I spend money on their food.
Phenomenon
Over the past decade, a decreasing percentage of the money was spent on feeding the Good Dog while an increasing percentage of the money was spent on feeding the Bad Dog.
Conclusion
Therefore, I'm spending less money on the GD's food than a decade ago.
The Missing Assumption
The total amount of money spent on Dog Food has not increased.
Explanation
We don't know how much I spent on the food for each dog. Let's say I spent $950 for GD and $50 for BD a decade ago. My budget was $1,000. 95% for GD and 5% for BD. The conclusion is valid if my budge had stayed the same or shrunken.
But if my budget for dog food is now $10,000, and now I'm spending $5,000 (50%) for GD and $5,000 for BD (50%). Now we can see the percentage has changed from 95% to 10%, the amount of money spent on GD's food has increased $4,050, which makes the conclusion invalid.
Remember, the task is to find an assumption that makes the argument Valid.
P1: Phenomenon
- the association of medieval canon lawyers didn't enforce standards of conduct.
P2: Hypothesis
- 1. These canon lawyers were scrupulous (didn't violate the standards much)
- 2. Violations were common and went unpunished.
P3: Support for Hypothesis 2
- church court was not efficient at disciplining the lawyers (compare to the civil court)
P4: Additional support for Hypothesis 2
- there is historic evidence for the inefficiency.
P5: Consequence
- criticism mentioned in P4 induced self-defense of the canon lawyers.
For Q7, I chose D over E because I thought E could be bating me to assume "new immigrants? they must lower the average age!" of course it isn't but I thought it could be the trap especially given that it is attributing a change in the average age of the country's population to a change in the number of births, which will lower the average (like my LSAT score could lower the average LSAT score). But it wasn't about that. It's an analogy question, so "structure" is of utmost importance. Don't overthink.
Now, D is incorrect because it lacks a component. It only has TWO components: 1 input (number of false crime reports) and 1 output (average annual number of crimes). However, the original text requires THREE components: 2 inputs (volcanic eruption and El Nino) and 1 output (temperature change). That's why it's incorrect. On the other hand, E matches the structure for having 2 inputs (ages of new immigrants & a change in the number of births) and 1 output (change in the average age of the country's population).
According to the stimulus, "stars that are too cool to burn hydrogen" are the definition of Brown Dwarfs.
So if you eliminate AC B for seeing "hydrogen" alone, that would be a mistake.
If B is restated, it would be "Most brown dwarfs are too cool to destroy lithium completely"
What if there are some BD that still contain lithium? Our stimulus tells us only the coolest of the BD doesn't contain lithium. The argument still stands.
This is not a necessary assumption the argument requires, and that's why it is not a correct AC.